Derivative of continuous function exists if limit of derivative exists











up vote
6
down vote

favorite
1












I'm stuck on this old qualifier problem. I suppose one could do it using the basic definitions of continuity and differentiability, but is there a simpler way? (For example, using DCT, FTC, Lebesgue differentiation theorem, etc.)



Let $f:mathbb{R} mapsto mathbb{R}$ be continuous. Suppose $f$ is differentiable away from $0$ and lim$_{x to 0} f^prime(x)$ exists. Show $f^prime(0)$ exists.










share|cite|improve this question


























    up vote
    6
    down vote

    favorite
    1












    I'm stuck on this old qualifier problem. I suppose one could do it using the basic definitions of continuity and differentiability, but is there a simpler way? (For example, using DCT, FTC, Lebesgue differentiation theorem, etc.)



    Let $f:mathbb{R} mapsto mathbb{R}$ be continuous. Suppose $f$ is differentiable away from $0$ and lim$_{x to 0} f^prime(x)$ exists. Show $f^prime(0)$ exists.










    share|cite|improve this question
























      up vote
      6
      down vote

      favorite
      1









      up vote
      6
      down vote

      favorite
      1






      1





      I'm stuck on this old qualifier problem. I suppose one could do it using the basic definitions of continuity and differentiability, but is there a simpler way? (For example, using DCT, FTC, Lebesgue differentiation theorem, etc.)



      Let $f:mathbb{R} mapsto mathbb{R}$ be continuous. Suppose $f$ is differentiable away from $0$ and lim$_{x to 0} f^prime(x)$ exists. Show $f^prime(0)$ exists.










      share|cite|improve this question













      I'm stuck on this old qualifier problem. I suppose one could do it using the basic definitions of continuity and differentiability, but is there a simpler way? (For example, using DCT, FTC, Lebesgue differentiation theorem, etc.)



      Let $f:mathbb{R} mapsto mathbb{R}$ be continuous. Suppose $f$ is differentiable away from $0$ and lim$_{x to 0} f^prime(x)$ exists. Show $f^prime(0)$ exists.







      calculus real-analysis analysis






      share|cite|improve this question













      share|cite|improve this question











      share|cite|improve this question




      share|cite|improve this question










      asked Jul 5 '14 at 14:46









      StrangerLoop

      73649




      73649






















          2 Answers
          2






          active

          oldest

          votes

















          up vote
          9
          down vote



          accepted










          By the mean value theorem, there is a $c_xin (0,x)$ resp. $c_xin (x,0)$, depending on whether $x > 0$ or $x < 0$, such that



          $$frac{f(x)-f(0)}{x} = f'(c_x).$$



          As $xto 0$, by the squeeze lemma, also $c_xto 0$, hence



          $$lim_{xto 0} frac{f(x)-f(0)}{x} = lim_{xto 0}f'(c_x)$$



          exists.






          share|cite|improve this answer





















          • Nice answer! Thanks.
            – StrangerLoop
            Jul 5 '14 at 14:53


















          up vote
          0
          down vote













          Not a proof, but an example I use with my students...



          Consider the derivative of an absolute value function.
          f(x)=abs(x)



          The derivative is f(x)={1, x>0
          {-1, x<0



          Therefore, the function is continuous and differentiable, but the derivative is not continuous.






          share|cite|improve this answer





















          • The question assumes that the limit of the derivative exists. This is not the case here.
            – Arnaud D.
            Mar 28 at 14:30











          Your Answer





          StackExchange.ifUsing("editor", function () {
          return StackExchange.using("mathjaxEditing", function () {
          StackExchange.MarkdownEditor.creationCallbacks.add(function (editor, postfix) {
          StackExchange.mathjaxEditing.prepareWmdForMathJax(editor, postfix, [["$", "$"], ["\\(","\\)"]]);
          });
          });
          }, "mathjax-editing");

          StackExchange.ready(function() {
          var channelOptions = {
          tags: "".split(" "),
          id: "69"
          };
          initTagRenderer("".split(" "), "".split(" "), channelOptions);

          StackExchange.using("externalEditor", function() {
          // Have to fire editor after snippets, if snippets enabled
          if (StackExchange.settings.snippets.snippetsEnabled) {
          StackExchange.using("snippets", function() {
          createEditor();
          });
          }
          else {
          createEditor();
          }
          });

          function createEditor() {
          StackExchange.prepareEditor({
          heartbeatType: 'answer',
          convertImagesToLinks: true,
          noModals: true,
          showLowRepImageUploadWarning: true,
          reputationToPostImages: 10,
          bindNavPrevention: true,
          postfix: "",
          imageUploader: {
          brandingHtml: "Powered by u003ca class="icon-imgur-white" href="https://imgur.com/"u003eu003c/au003e",
          contentPolicyHtml: "User contributions licensed under u003ca href="https://creativecommons.org/licenses/by-sa/3.0/"u003ecc by-sa 3.0 with attribution requiredu003c/au003e u003ca href="https://stackoverflow.com/legal/content-policy"u003e(content policy)u003c/au003e",
          allowUrls: true
          },
          noCode: true, onDemand: true,
          discardSelector: ".discard-answer"
          ,immediatelyShowMarkdownHelp:true
          });


          }
          });














          draft saved

          draft discarded


















          StackExchange.ready(
          function () {
          StackExchange.openid.initPostLogin('.new-post-login', 'https%3a%2f%2fmath.stackexchange.com%2fquestions%2f857139%2fderivative-of-continuous-function-exists-if-limit-of-derivative-exists%23new-answer', 'question_page');
          }
          );

          Post as a guest















          Required, but never shown

























          2 Answers
          2






          active

          oldest

          votes








          2 Answers
          2






          active

          oldest

          votes









          active

          oldest

          votes






          active

          oldest

          votes








          up vote
          9
          down vote



          accepted










          By the mean value theorem, there is a $c_xin (0,x)$ resp. $c_xin (x,0)$, depending on whether $x > 0$ or $x < 0$, such that



          $$frac{f(x)-f(0)}{x} = f'(c_x).$$



          As $xto 0$, by the squeeze lemma, also $c_xto 0$, hence



          $$lim_{xto 0} frac{f(x)-f(0)}{x} = lim_{xto 0}f'(c_x)$$



          exists.






          share|cite|improve this answer





















          • Nice answer! Thanks.
            – StrangerLoop
            Jul 5 '14 at 14:53















          up vote
          9
          down vote



          accepted










          By the mean value theorem, there is a $c_xin (0,x)$ resp. $c_xin (x,0)$, depending on whether $x > 0$ or $x < 0$, such that



          $$frac{f(x)-f(0)}{x} = f'(c_x).$$



          As $xto 0$, by the squeeze lemma, also $c_xto 0$, hence



          $$lim_{xto 0} frac{f(x)-f(0)}{x} = lim_{xto 0}f'(c_x)$$



          exists.






          share|cite|improve this answer





















          • Nice answer! Thanks.
            – StrangerLoop
            Jul 5 '14 at 14:53













          up vote
          9
          down vote



          accepted







          up vote
          9
          down vote



          accepted






          By the mean value theorem, there is a $c_xin (0,x)$ resp. $c_xin (x,0)$, depending on whether $x > 0$ or $x < 0$, such that



          $$frac{f(x)-f(0)}{x} = f'(c_x).$$



          As $xto 0$, by the squeeze lemma, also $c_xto 0$, hence



          $$lim_{xto 0} frac{f(x)-f(0)}{x} = lim_{xto 0}f'(c_x)$$



          exists.






          share|cite|improve this answer












          By the mean value theorem, there is a $c_xin (0,x)$ resp. $c_xin (x,0)$, depending on whether $x > 0$ or $x < 0$, such that



          $$frac{f(x)-f(0)}{x} = f'(c_x).$$



          As $xto 0$, by the squeeze lemma, also $c_xto 0$, hence



          $$lim_{xto 0} frac{f(x)-f(0)}{x} = lim_{xto 0}f'(c_x)$$



          exists.







          share|cite|improve this answer












          share|cite|improve this answer



          share|cite|improve this answer










          answered Jul 5 '14 at 14:50









          Daniel Fischer

          173k16159281




          173k16159281












          • Nice answer! Thanks.
            – StrangerLoop
            Jul 5 '14 at 14:53


















          • Nice answer! Thanks.
            – StrangerLoop
            Jul 5 '14 at 14:53
















          Nice answer! Thanks.
          – StrangerLoop
          Jul 5 '14 at 14:53




          Nice answer! Thanks.
          – StrangerLoop
          Jul 5 '14 at 14:53










          up vote
          0
          down vote













          Not a proof, but an example I use with my students...



          Consider the derivative of an absolute value function.
          f(x)=abs(x)



          The derivative is f(x)={1, x>0
          {-1, x<0



          Therefore, the function is continuous and differentiable, but the derivative is not continuous.






          share|cite|improve this answer





















          • The question assumes that the limit of the derivative exists. This is not the case here.
            – Arnaud D.
            Mar 28 at 14:30















          up vote
          0
          down vote













          Not a proof, but an example I use with my students...



          Consider the derivative of an absolute value function.
          f(x)=abs(x)



          The derivative is f(x)={1, x>0
          {-1, x<0



          Therefore, the function is continuous and differentiable, but the derivative is not continuous.






          share|cite|improve this answer





















          • The question assumes that the limit of the derivative exists. This is not the case here.
            – Arnaud D.
            Mar 28 at 14:30













          up vote
          0
          down vote










          up vote
          0
          down vote









          Not a proof, but an example I use with my students...



          Consider the derivative of an absolute value function.
          f(x)=abs(x)



          The derivative is f(x)={1, x>0
          {-1, x<0



          Therefore, the function is continuous and differentiable, but the derivative is not continuous.






          share|cite|improve this answer












          Not a proof, but an example I use with my students...



          Consider the derivative of an absolute value function.
          f(x)=abs(x)



          The derivative is f(x)={1, x>0
          {-1, x<0



          Therefore, the function is continuous and differentiable, but the derivative is not continuous.







          share|cite|improve this answer












          share|cite|improve this answer



          share|cite|improve this answer










          answered Mar 28 at 12:35









          user546542

          1




          1












          • The question assumes that the limit of the derivative exists. This is not the case here.
            – Arnaud D.
            Mar 28 at 14:30


















          • The question assumes that the limit of the derivative exists. This is not the case here.
            – Arnaud D.
            Mar 28 at 14:30
















          The question assumes that the limit of the derivative exists. This is not the case here.
          – Arnaud D.
          Mar 28 at 14:30




          The question assumes that the limit of the derivative exists. This is not the case here.
          – Arnaud D.
          Mar 28 at 14:30


















          draft saved

          draft discarded




















































          Thanks for contributing an answer to Mathematics Stack Exchange!


          • Please be sure to answer the question. Provide details and share your research!

          But avoid



          • Asking for help, clarification, or responding to other answers.

          • Making statements based on opinion; back them up with references or personal experience.


          Use MathJax to format equations. MathJax reference.


          To learn more, see our tips on writing great answers.





          Some of your past answers have not been well-received, and you're in danger of being blocked from answering.


          Please pay close attention to the following guidance:


          • Please be sure to answer the question. Provide details and share your research!

          But avoid



          • Asking for help, clarification, or responding to other answers.

          • Making statements based on opinion; back them up with references or personal experience.


          To learn more, see our tips on writing great answers.




          draft saved


          draft discarded














          StackExchange.ready(
          function () {
          StackExchange.openid.initPostLogin('.new-post-login', 'https%3a%2f%2fmath.stackexchange.com%2fquestions%2f857139%2fderivative-of-continuous-function-exists-if-limit-of-derivative-exists%23new-answer', 'question_page');
          }
          );

          Post as a guest















          Required, but never shown





















































          Required, but never shown














          Required, but never shown












          Required, but never shown







          Required, but never shown

































          Required, but never shown














          Required, but never shown












          Required, but never shown







          Required, but never shown







          Popular posts from this blog

          Different font size/position of beamer's navigation symbols template's content depending on regular/plain...

          Berounka

          I want to find a topological embedding $f : X rightarrow Y$ and $g: Y rightarrow X$, yet $X$ is not...